Search
 
 
  Engleski
 
 
 
Open in this window (click to change)
Forum@DeGiorgi: Početna
Forum za podršku nastavi na PMF-MO
Login Registracija FAQ Smajlići Članstvo Pretražnik Forum@DeGiorgi: Početna

Pomoć oko zadatka
WWW:
Idite na Prethodno  1, 2, 3, 4, 5  Sljedeće
Moja sarma
 
Započnite novu temu   Odgovorite na temu   printer-friendly view    Forum@DeGiorgi: Početna -> Kolegiji 3. godine -> Mjera i integral
Prethodna tema :: Sljedeća tema  
Autor/ica Poruka
Lafiel
Forumaš(ica)
Forumaš(ica)


Pridružen/a: 26. 09. 2007. (09:56:59)
Postovi: (153)16
Spol: žensko
Sarma = la pohva - posuda
27 = 51 - 24

PostPostano: 20:48 čet, 11. 4. 2013    Naslov: Citirajte i odgovorite

Hvala ti, Phoenix. Da mogu, dala bih ti još 6 karma bodova. :)

[size=9][color=#999999]Added after 6 minutes:[/color][/size]

[quote="kosani"]2 pitanja:

1) 4. zadatak b) http://web.math.pmf.unizg.hr/nastava/mii/files/mi-kol1-2009-rj.pdf

Kako znamo da [tex]E \in M(R)[/tex]. Pitam jer ne razumijem kako možemo zaključiti da [tex]E \cup F_{i} \in M(R)[/tex][/quote]
Ako imaš rastući niz [tex](F_{i})[/tex] iz [tex]\mathcal{L}(E)[/tex], onda je [tex](E \cup F_{i}) \in \mathcal{M}(R)[/tex] po samoj definiciji skupa [tex]\mathcal{L}(E)[/tex], a još k tome je i rastući jer je [tex](F_{i})[/tex] rastući.

@Phoenix, nigdje nije rečeno da je [tex]E[/tex] iz [tex]\mathcal{R}[/tex], samo da je [tex]E \subseteq X[/tex]. Osim ako nekim drugim načinom nisi došao do zaključka da je iz [tex]\mathcal{R}[/tex]. Pričamo o b) dijelu zadatka. :)
Hvala ti, Phoenix. Da mogu, dala bih ti još 6 karma bodova. Smile

Added after 6 minutes:

kosani (napisa):
2 pitanja:

1) 4. zadatak b) http://web.math.pmf.unizg.hr/nastava/mii/files/mi-kol1-2009-rj.pdf

Kako znamo da [tex]E \in M(R)[/tex]. Pitam jer ne razumijem kako možemo zaključiti da [tex]E \cup F_{i} \in M(R)[/tex]

Ako imaš rastući niz [tex](F_{i})[/tex] iz [tex]\mathcal{L}(E)[/tex], onda je [tex](E \cup F_{i}) \in \mathcal{M}(R)[/tex] po samoj definiciji skupa [tex]\mathcal{L}(E)[/tex], a još k tome je i rastući jer je [tex](F_{i})[/tex] rastući.

@Phoenix, nigdje nije rečeno da je [tex]E[/tex] iz [tex]\mathcal{R}[/tex], samo da je [tex]E \subseteq X[/tex]. Osim ako nekim drugim načinom nisi došao do zaključka da je iz [tex]\mathcal{R}[/tex]. Pričamo o b) dijelu zadatka. Smile



_________________
Weit von hier fällt Gold von den Sternen
[Vrh]
Korisnički profil Pošaljite privatnu poruku
Phoenix
Forumaš(ica)
Forumaš(ica)


Pridružen/a: 15. 05. 2010. (18:46:07)
Postovi: (164)16
Sarma: -

PostPostano: 20:55 čet, 11. 4. 2013    Naslov: Citirajte i odgovorite

Hvala, Lafiel! Ja čitam zadatak pod [tex]c)[/tex] i, iskreno, bilo mi je malo sumnjivo... :P
Hvala, Lafiel! Ja čitam zadatak pod [tex]c)[/tex] i, iskreno, bilo mi je malo sumnjivo... Razz


[Vrh]
Korisnički profil Pošaljite privatnu poruku
kosani
Forumaš(ica)
Forumaš(ica)


Pridružen/a: 14. 11. 2010. (21:22:58)
Postovi: (26)16
Sarma = la pohva - posuda
= 9 - 7

PostPostano: 21:51 čet, 11. 4. 2013    Naslov: Citirajte i odgovorite

[quote="Phoenix"]
kosani,

[tex]E \in \mathcal{R}[/tex] pa je ...
[/quote]

gdje imamo da [tex]E \in \mathcal{R}[/tex] ? samo imamo da
[tex]E \subset \mathcal{X}[/tex]
Phoenix (napisa):

kosani,

[tex]E \in \mathcal{R}[/tex] pa je ...


gdje imamo da [tex]E \in \mathcal{R}[/tex] ? samo imamo da
[tex]E \subset \mathcal{X}[/tex]


[Vrh]
Korisnički profil Pošaljite privatnu poruku
Phoenix
Forumaš(ica)
Forumaš(ica)


Pridružen/a: 15. 05. 2010. (18:46:07)
Postovi: (164)16
Sarma: -

PostPostano: 22:20 čet, 11. 4. 2013    Naslov: Citirajte i odgovorite

[quote="kosani"][quote="Phoenix"]
kosani,

[tex]E \in \mathcal{R}[/tex] pa je ...
[/quote]

gdje imamo da [tex]E \in \mathcal{R}[/tex] ? samo imamo da
[tex]E \subset \mathcal{X}[/tex][/quote]

Oprosti, ignoriraj tu grešku. :P Mislio sam da pitaš za [tex]c)[/tex] zadatak pa sam ti zato to napisao.
S druge strane, Lafiel ti je ponudila točno rješenje! :) Pogledaj dva posta prije ovog svog. ;)
kosani (napisa):
Phoenix (napisa):

kosani,

[tex]E \in \mathcal{R}[/tex] pa je ...


gdje imamo da [tex]E \in \mathcal{R}[/tex] ? samo imamo da
[tex]E \subset \mathcal{X}[/tex]


Oprosti, ignoriraj tu grešku. Razz Mislio sam da pitaš za [tex]c)[/tex] zadatak pa sam ti zato to napisao.
S druge strane, Lafiel ti je ponudila točno rješenje! Smile Pogledaj dva posta prije ovog svog. Wink


[Vrh]
Korisnički profil Pošaljite privatnu poruku
kosani
Forumaš(ica)
Forumaš(ica)


Pridružen/a: 14. 11. 2010. (21:22:58)
Postovi: (26)16
Sarma = la pohva - posuda
= 9 - 7

PostPostano: 22:31 čet, 11. 4. 2013    Naslov: Citirajte i odgovorite

Hvala vam oboje!
Hvala vam oboje!


[Vrh]
Korisnički profil Pošaljite privatnu poruku
kobila krsto
Forumaš(ica)
Forumaš(ica)


Pridružen/a: 02. 07. 2009. (16:55:08)
Postovi: (6A)16
Sarma = la pohva - posuda
-2 = 16 - 18

PostPostano: 22:34 pet, 31. 5. 2013    Naslov: Citirajte i odgovorite

****

EDIT : ups, ovdje je bio zadatak koji je postojao na vježbama ( kopirao sam lanjske pa nisam vidio od ove godine ). pardon :)
****

EDIT : ups, ovdje je bio zadatak koji je postojao na vježbama ( kopirao sam lanjske pa nisam vidio od ove godine ). pardon Smile


[Vrh]
Korisnički profil Pošaljite privatnu poruku
Lafiel
Forumaš(ica)
Forumaš(ica)


Pridružen/a: 26. 09. 2007. (09:56:59)
Postovi: (153)16
Spol: žensko
Sarma = la pohva - posuda
27 = 51 - 24

PostPostano: 18:12 uto, 4. 6. 2013    Naslov: Citirajte i odgovorite

Ovo vjerojatno više spada pod Analizu nego pod Mjeru, ali pitanjce vezano uz [url=http://web.math.pmf.unizg.hr/nastava/mii/files/kviz11.pdf]Kviz 11 c)[/url]: u kakvom su odnosu [tex]\int_0^1 (- ln x)^p\,dx[/tex] i [tex]\int_0^1 \frac{dx}{\sqrt{x}}[/tex] ? :oops:
Ovo vjerojatno više spada pod Analizu nego pod Mjeru, ali pitanjce vezano uz Kviz 11 c): u kakvom su odnosu [tex]\int_0^1 (- ln x)^p\,dx[/tex] i [tex]\int_0^1 \frac{dx}{\sqrt{x}}[/tex] ? Embarassed



_________________
Weit von hier fällt Gold von den Sternen
[Vrh]
Korisnički profil Pošaljite privatnu poruku
mathh5
Forumaš(ica)
Forumaš(ica)


Pridružen/a: 26. 05. 2012. (12:16:25)
Postovi: (E)16
Sarma = la pohva - posuda
-2 = 1 - 3

PostPostano: 14:05 čet, 6. 6. 2013    Naslov: Citirajte i odgovorite

Pitanje iz prošlogodišnjeg kolokvija http://web.math.pmf.unizg.hr/nastava/mii/files/mii-kol2-2012-rj.pdf
4. zadatak (b) Zašto na kraju kažemo da je ||f||p < +beskonačno <=> (p/4)-2<-1
Kako smo došli do ove posljednje nejednakosti? zašto -1?
Unaprijed hvala!
Pitanje iz prošlogodišnjeg kolokvija http://web.math.pmf.unizg.hr/nastava/mii/files/mii-kol2-2012-rj.pdf
4. zadatak (b) Zašto na kraju kažemo da je ||f||p < +beskonačno <=> (p/4)-2<-1
Kako smo došli do ove posljednje nejednakosti? zašto -1?
Unaprijed hvala!


[Vrh]
Korisnički profil Pošaljite privatnu poruku
ddujmic
Forumaš(ica)
Forumaš(ica)


Pridružen/a: 17. 02. 2009. (14:01:31)
Postovi: (75)16
Sarma = la pohva - posuda
= 7 - 1

PostPostano: 14:22 čet, 6. 6. 2013    Naslov: Citirajte i odgovorite

Stavis x^-1.. pa onda znas da to konvegrira akko -p/4 +2>1.. pa dobijes da je p<4.. a gore smo uzeli da je p>0.. pa je onda p iz <0,4>
Stavis x^-1.. pa onda znas da to konvegrira akko -p/4 +2>1.. pa dobijes da je p<4.. a gore smo uzeli da je p>0.. pa je onda p iz <0,4>



_________________
Nothing lasts forever
Even cold November rain
[Vrh]
Korisnički profil Pošaljite privatnu poruku MSNM
ceps
Forumaš(ica)
Forumaš(ica)


Pridružen/a: 08. 10. 2010. (13:03:07)
Postovi: (13A)16
Sarma = la pohva - posuda
71 = 74 - 3

PostPostano: 15:49 čet, 6. 6. 2013    Naslov: Citirajte i odgovorite

[quote="Lafiel"]Ovo vjerojatno više spada pod Analizu nego pod Mjeru, ali pitanjce vezano uz [url=http://web.math.pmf.unizg.hr/nastava/mii/files/kviz11.pdf]Kviz 11 c)[/url]: u kakvom su odnosu [tex]\int_0^1 (- ln x)^p\,dx[/tex] i [tex]\int_0^1 \frac{dx}{\sqrt{x}}[/tex] ? :oops:[/quote]

Pa, fora je u uspoređivanju funkcija ispod integrala.
Pomoću D'Alambertove formule možeš vidjeti da [latex]\lim_{x \to 0} \frac{-ln(x)}{\frac{1}{\sqrt{x}}} = 0[/latex] . Kad imaš to, možeš sličnom forom zaključiti isto za [latex]|ln(x)|^p[/latex]. Preko toga onda možeš ograničiti integral od logaritma integralom od korijena.

Možda netko ima neku bržu/lakšu/pametniju foru, to meni pada na pamet.
Lafiel (napisa):
Ovo vjerojatno više spada pod Analizu nego pod Mjeru, ali pitanjce vezano uz Kviz 11 c): u kakvom su odnosu [tex]\int_0^1 (- ln x)^p\,dx[/tex] i [tex]\int_0^1 \frac{dx}{\sqrt{x}}[/tex] ? Embarassed


Pa, fora je u uspoređivanju funkcija ispod integrala.
Pomoću D'Alambertove formule možeš vidjeti da . Kad imaš to, možeš sličnom forom zaključiti isto za . Preko toga onda možeš ograničiti integral od logaritma integralom od korijena.

Možda netko ima neku bržu/lakšu/pametniju foru, to meni pada na pamet.


[Vrh]
Korisnički profil Pošaljite privatnu poruku
Gost






PostPostano: 0:21 pet, 7. 6. 2013    Naslov: Citirajte i odgovorite

[quote="ceps"][quote="Lafiel"]Ovo vjerojatno više spada pod Analizu nego pod Mjeru, ali pitanjce vezano uz [url=http://web.math.pmf.unizg.hr/nastava/mii/files/kviz11.pdf]Kviz 11 c)[/url]: u kakvom su odnosu [tex]\int_0^1 (- ln x)^p\,dx[/tex] i [tex]\int_0^1 \frac{dx}{\sqrt{x}}[/tex] ? :oops:[/quote]

Pa, fora je u uspoređivanju funkcija ispod integrala.
Pomoću D'Alambertove formule možeš vidjeti da [latex]\lim_{x \to 0} \frac{-ln(x)}{\frac{1}{\sqrt{x}}} = 0[/latex] . Kad imaš to, možeš sličnom forom zaključiti isto za [latex]|ln(x)|^p[/latex]. Preko toga onda možeš ograničiti integral od logaritma integralom od korijena.

Možda netko ima neku bržu/lakšu/pametniju foru, to meni pada na pamet.[/quote]

Mozes li ovo detaljnije raspisati? Trazio sam formulu o kojoj pricas no nisam ju nasao.
Hvala
ceps (napisa):
Lafiel (napisa):
Ovo vjerojatno više spada pod Analizu nego pod Mjeru, ali pitanjce vezano uz Kviz 11 c): u kakvom su odnosu [tex]\int_0^1 (- ln x)^p\,dx[/tex] i [tex]\int_0^1 \frac{dx}{\sqrt{x}}[/tex] ? Embarassed


Pa, fora je u uspoređivanju funkcija ispod integrala.
Pomoću D'Alambertove formule možeš vidjeti da . Kad imaš to, možeš sličnom forom zaključiti isto za . Preko toga onda možeš ograničiti integral od logaritma integralom od korijena.

Možda netko ima neku bržu/lakšu/pametniju foru, to meni pada na pamet.


Mozes li ovo detaljnije raspisati? Trazio sam formulu o kojoj pricas no nisam ju nasao.
Hvala


[Vrh]
Lafiel
Forumaš(ica)
Forumaš(ica)


Pridružen/a: 26. 09. 2007. (09:56:59)
Postovi: (153)16
Spol: žensko
Sarma = la pohva - posuda
27 = 51 - 24

PostPostano: 19:26 pon, 17. 6. 2013    Naslov: Citirajte i odgovorite

Bi li mogao netko malo razjasniti dokaz Teorema 5.15. na 33. stranici [url=http://web.math.pmf.unizg.hr/nastava/mii/files/mii_predavanja.pdf]skripte[/url]?

Malo me muči ovo "lako se vidi da je [tex]f_1 \leq f_2 \leq ...[/tex] i [tex]f_n(x) \nearrow f(x)[/tex]" tj. ja to baš ne vidim, pogotovo ne lako. :-k
Bi li mogao netko malo razjasniti dokaz Teorema 5.15. na 33. stranici skripte?

Malo me muči ovo "lako se vidi da je [tex]f_1 \leq f_2 \leq ...[/tex] i [tex]f_n(x) \nearrow f(x)[/tex]" tj. ja to baš ne vidim, pogotovo ne lako. Think



_________________
Weit von hier fällt Gold von den Sternen
[Vrh]
Korisnički profil Pošaljite privatnu poruku
Gost






PostPostano: 22:22 pon, 17. 6. 2013    Naslov: Citirajte i odgovorite

Da konvergira prema f se vidi lako iz definicije:
1) ako je f(x) = +beskonacno onda je fn(x) = n pa je sad ocito
2) ako je f(x) konacan, onda ce se za dovoljno veliki n, fn(x) razlikovati od f(x) za 1/2^n (vidi se iz definicije A(n,k)) pa ocito konvergira prema f(x)

Da raste: ako je f(x) = +beskonacno, onda je fn(x) = n pa ocito raste.
Neka je f(x) konacan. Neka je (najvece cijelo od f(x)) = m. Tada za n<m je fn(x) = n pa ocito raste. Neka je sada n>=m proizvoljan, tada po definiciji od fn(x) postoji k takav da

(k-1)/2^n <= f(x) < k/2^n

pa je

fn(x) = (k-1)/2^n ; posebno fn(x)=(2k-2)/2^(n+1)

sada ocito f(n+1)(x) moze biti po definiciji:

(2k-2)/2^(n+1) ili (2k-1)/2^(n+1),

jer je f(x) iz

(2k-2)/2^(n+1) <= f(x) < (2k-1)/2^(n+1)
ili
(2k-1)/2^(n+1) <= f(x) < (2k)/2^(n+1)

Zao mi je sto nije u latexu :oops:
Da konvergira prema f se vidi lako iz definicije:
1) ako je f(x) = +beskonacno onda je fn(x) = n pa je sad ocito
2) ako je f(x) konacan, onda ce se za dovoljno veliki n, fn(x) razlikovati od f(x) za 1/2^n (vidi se iz definicije A(n,k)) pa ocito konvergira prema f(x)

Da raste: ako je f(x) = +beskonacno, onda je fn(x) = n pa ocito raste.
Neka je f(x) konacan. Neka je (najvece cijelo od f(x)) = m. Tada za n<m je fn(x) = n pa ocito raste. Neka je sada n>=m proizvoljan, tada po definiciji od fn(x) postoji k takav da

(k-1)/2^n <= f(x) < k/2^n

pa je

fn(x) = (k-1)/2^n ; posebno fn(x)=(2k-2)/2^(n+1)

sada ocito f(n+1)(x) moze biti po definiciji:

(2k-2)/2^(n+1) ili (2k-1)/2^(n+1),

jer je f(x) iz

(2k-2)/2^(n+1) <= f(x) < (2k-1)/2^(n+1)
ili
(2k-1)/2^(n+1) <= f(x) < (2k)/2^(n+1)

Zao mi je sto nije u latexu Embarassed


[Vrh]
Phoenix
Forumaš(ica)
Forumaš(ica)


Pridružen/a: 15. 05. 2010. (18:46:07)
Postovi: (164)16
Sarma: -

PostPostano: 22:53 pon, 17. 6. 2013    Naslov: Citirajte i odgovorite

Da, pogotovo kada se u kolokviju vjerojatno nije smjelo "lako vidjeti"... :)

Možda ne bi bilo loše znati otprilike kako točno funkcije [tex]f_n[/tex] opisuju funkciju [tex]f[/tex]. :)
OK, za neki [tex]n[/tex] funkcija [tex]f_n[/tex] je jednostavna funkcija i poprima vrijednosti iz skupa [tex]\left\{ 0, \frac{1}{2^n}, ..., \frac{n \cdot 2^n-1}{2^n}, \frac{n \cdot 2^n}{2^n} = n \right\}[/tex] i to na sljedeći način: ako je (za neki [tex]x \in X[/tex]) vrijednost [tex]f(x)[/tex] u segmentu [tex]\left[ \frac{k-1}{2^n}, \frac{k}{2^n} \right>[/tex], tada je [tex]f_n(x)=\frac{k-1}{2^n}[/tex]. (I, dodatno, ako je [tex]f(x)\geq n[/tex], tada je jednostavno [tex]f_n(x)=n[/tex].)
Evo primjer kako otprilike izgleda funkcija [tex]f_n[/tex] za neku [tex]f[/tex] koja je nalik krivulji polinoma trećeg stupnja:

[img]http://i43.tinypic.com/357hd77.png[/img]

Dakle, ova valovita krivulja neka je promatrana funkcija [tex]f[/tex], ove svjetloplave crte neka označavaju vrijednosti oblika [tex]\frac{k}{2^n}[/tex]. Tada tamnoplave crte zajedno predstavljaju graf funkcije [tex]f_n[/tex]!
Sada, prijelaz na [tex]f_{n+1}[/tex]... Direktno iz definicije, ta funkcija postiže vrijednosti [tex]0, \frac{1}{2^{n+1}}, \frac{2}{2^{n+1}}=\frac{1}{2^n}, \frac{3}{2^{n+1}}, \frac{4}{2^{n+1}}=\frac{2}{2^n}, ..., \frac{(n+1) \cdot 2^{n+1}-1}{2^{n+1}}, \frac{(n+1) \cdot 2^{n+1}}{2^{n+1}}=n+1[/tex]. Odnosno, u odnosu na prethodnu [tex]f_n[/tex], između svake dvije svjetloplave crte nadodaješ još jednu svjetloplavu crtu (kao i između one najdonje te [tex]x[/tex] osi :) ), a onda između [tex]n[/tex] i [tex]n+1[/tex] dodaš onoliko jednako udaljenih svjetloplavih crta koliko je potrebno (da mreža bude podjednaka). I dobivaš sljedeće:

[img]http://i39.tinypic.com/fbj6sh.png[/img]

Sada je nekako jasnije da ovaj niz funkcija poprilično dobro opisuje početnu funkciju [tex]f[/tex]. :)

A sada, vezano uz tvoje pitanje, uoči na ovim dvijema skicama sljedeće: pri prijelazu s jedne slike na drugu, određeni komad tamnoplave linije ili ostaje na istom mjestu ili se podiže, i to za točno [tex]\frac{1}{2^{n+1}}[/tex].

Ovo zapažanje ti pomaže da argumentiraš monotonost i konvergenciju niza funkcija i to na sljedeći način:
- Za [tex]x \in X[/tex] i [tex]n \in \mathbb{N}[/tex] postoji [tex]k \in \mathbb{N}[/tex] takav da je [tex]\frac{k-1}{2^n} \leq f(x) < \frac{k}{2^n}[/tex], iz čega onda slijedi [tex]f_n(x)=\frac{k-1}{2^n}[/tex]. Ovisno o tome je li [tex]f(x)<\frac{2k-1}{2^{n+1}}[/tex] ili [tex]f(x) \geq \frac{2k-1}{2^{n+1}}[/tex], slijedi da je [tex]f_{n+1}(x)=\frac{k-1}{2^n}[/tex] ili [tex]f_{n+1}(x)=\frac{2k-1}{2^{n+1}}[/tex]. U svakom slučaju je [tex]f_n(x) \leq f_{n+1}(x)[/tex]. Slična argumentacija slijedi i za [tex]f(x) \geq n[/tex] (čisto zbog formalnosti drugačiji slučaj).
- U slučaju [tex]f(x)<+\infty[/tex] (inače je [tex]f_n(x)=n \stackrel{n \rightarrow +\infty}{\longrightarrow} +\infty[/tex]) postoji dovoljno veliki [tex]n \in \mathbb{N}[/tex] takav da je [tex]n \leq f(x) < n+1[/tex]. Po definiciji od [tex]f_n[/tex] je tada [tex]0 \leq f(x)-f_n(x) \leq \frac{1}{2^{n+1}} \stackrel{n \rightarrow +\infty}{\longrightarrow} 0[/tex]. Iz ovoga slijedi [tex]\lim \limits_{n \rightarrow +\infty}f_n(x) = f(x), \forall x \in X[/tex].


Evo, ovo je moj komentar na teorem, zapravo slično onome što je kolega Gost već napisao. :)
U principu, po meni je najbitnije imati u glavi ove skice (koje su mogle biti i bolje i detaljnije, ali eto, znam crtati samo u Paintu i nemam skener :D ) i onda praktički znaš "pjevati" dokaz (nemaš problema niti s definicijom skupova [tex]A_n[/tex]). ;)
Da, pogotovo kada se u kolokviju vjerojatno nije smjelo "lako vidjeti"... Smile

Možda ne bi bilo loše znati otprilike kako točno funkcije [tex]f_n[/tex] opisuju funkciju [tex]f[/tex]. Smile
OK, za neki [tex]n[/tex] funkcija [tex]f_n[/tex] je jednostavna funkcija i poprima vrijednosti iz skupa [tex]\left\{ 0, \frac{1}{2^n}, ..., \frac{n \cdot 2^n-1}{2^n}, \frac{n \cdot 2^n}{2^n} = n \right\}[/tex] i to na sljedeći način: ako je (za neki [tex]x \in X[/tex]) vrijednost [tex]f(x)[/tex] u segmentu [tex]\left[ \frac{k-1}{2^n}, \frac{k}{2^n} \right>[/tex], tada je [tex]f_n(x)=\frac{k-1}{2^n}[/tex]. (I, dodatno, ako je [tex]f(x)\geq n[/tex], tada je jednostavno [tex]f_n(x)=n[/tex].)
Evo primjer kako otprilike izgleda funkcija [tex]f_n[/tex] za neku [tex]f[/tex] koja je nalik krivulji polinoma trećeg stupnja:



Dakle, ova valovita krivulja neka je promatrana funkcija [tex]f[/tex], ove svjetloplave crte neka označavaju vrijednosti oblika [tex]\frac{k}{2^n}[/tex]. Tada tamnoplave crte zajedno predstavljaju graf funkcije [tex]f_n[/tex]!
Sada, prijelaz na [tex]f_{n+1}[/tex]... Direktno iz definicije, ta funkcija postiže vrijednosti [tex]0, \frac{1}{2^{n+1}}, \frac{2}{2^{n+1}}=\frac{1}{2^n}, \frac{3}{2^{n+1}}, \frac{4}{2^{n+1}}=\frac{2}{2^n}, ..., \frac{(n+1) \cdot 2^{n+1}-1}{2^{n+1}}, \frac{(n+1) \cdot 2^{n+1}}{2^{n+1}}=n+1[/tex]. Odnosno, u odnosu na prethodnu [tex]f_n[/tex], između svake dvije svjetloplave crte nadodaješ još jednu svjetloplavu crtu (kao i između one najdonje te [tex]x[/tex] osi Smile ), a onda između [tex]n[/tex] i [tex]n+1[/tex] dodaš onoliko jednako udaljenih svjetloplavih crta koliko je potrebno (da mreža bude podjednaka). I dobivaš sljedeće:



Sada je nekako jasnije da ovaj niz funkcija poprilično dobro opisuje početnu funkciju [tex]f[/tex]. Smile

A sada, vezano uz tvoje pitanje, uoči na ovim dvijema skicama sljedeće: pri prijelazu s jedne slike na drugu, određeni komad tamnoplave linije ili ostaje na istom mjestu ili se podiže, i to za točno [tex]\frac{1}{2^{n+1}}[/tex].

Ovo zapažanje ti pomaže da argumentiraš monotonost i konvergenciju niza funkcija i to na sljedeći način:
- Za [tex]x \in X[/tex] i [tex]n \in \mathbb{N}[/tex] postoji [tex]k \in \mathbb{N}[/tex] takav da je [tex]\frac{k-1}{2^n} \leq f(x) < \frac{k}{2^n}[/tex], iz čega onda slijedi [tex]f_n(x)=\frac{k-1}{2^n}[/tex]. Ovisno o tome je li [tex]f(x)<\frac{2k-1}{2^{n+1}}[/tex] ili [tex]f(x) \geq \frac{2k-1}{2^{n+1}}[/tex], slijedi da je [tex]f_{n+1}(x)=\frac{k-1}{2^n}[/tex] ili [tex]f_{n+1}(x)=\frac{2k-1}{2^{n+1}}[/tex]. U svakom slučaju je [tex]f_n(x) \leq f_{n+1}(x)[/tex]. Slična argumentacija slijedi i za [tex]f(x) \geq n[/tex] (čisto zbog formalnosti drugačiji slučaj).
- U slučaju [tex]f(x)<+\infty[/tex] (inače je [tex]f_n(x)=n \stackrel{n \rightarrow +\infty}{\longrightarrow} +\infty[/tex]) postoji dovoljno veliki [tex]n \in \mathbb{N}[/tex] takav da je [tex]n \leq f(x) < n+1[/tex]. Po definiciji od [tex]f_n[/tex] je tada [tex]0 \leq f(x)-f_n(x) \leq \frac{1}{2^{n+1}} \stackrel{n \rightarrow +\infty}{\longrightarrow} 0[/tex]. Iz ovoga slijedi [tex]\lim \limits_{n \rightarrow +\infty}f_n(x) = f(x), \forall x \in X[/tex].


Evo, ovo je moj komentar na teorem, zapravo slično onome što je kolega Gost već napisao. Smile
U principu, po meni je najbitnije imati u glavi ove skice (koje su mogle biti i bolje i detaljnije, ali eto, znam crtati samo u Paintu i nemam skener Very Happy ) i onda praktički znaš "pjevati" dokaz (nemaš problema niti s definicijom skupova [tex]A_n[/tex]). Wink


[Vrh]
Korisnički profil Pošaljite privatnu poruku
Lafiel
Forumaš(ica)
Forumaš(ica)


Pridružen/a: 26. 09. 2007. (09:56:59)
Postovi: (153)16
Spol: žensko
Sarma = la pohva - posuda
27 = 51 - 24

PostPostano: 17:56 uto, 18. 6. 2013    Naslov: Citirajte i odgovorite

Hvala vam obojici, pogotovo Phoenixu na trudu s crtežom. :bow: :bow: :bow:
Hvala vam obojici, pogotovo Phoenixu na trudu s crtežom. I bow before you I bow before you I bow before you



_________________
Weit von hier fällt Gold von den Sternen
[Vrh]
Korisnički profil Pošaljite privatnu poruku
Gost






PostPostano: 10:33 sri, 19. 6. 2013    Naslov: Citirajte i odgovorite

http://web.math.pmf.unizg.hr/nastava/mii/files/mii_2013_kol2_rj.pdf

Nas kolokvij, ZAD 4, (b)

Moze li objasnjenje cijelog zadatka, tj. dio gdje smo trazili dominantnu fju, zasto se x gledao po intervalima te kako smo zakljucili da je integral od g(x) konacan? Koje su to tvrdnje koje su spomenute u rjesenjima?
http://web.math.pmf.unizg.hr/nastava/mii/files/mii_2013_kol2_rj.pdf

Nas kolokvij, ZAD 4, (b)

Moze li objasnjenje cijelog zadatka, tj. dio gdje smo trazili dominantnu fju, zasto se x gledao po intervalima te kako smo zakljucili da je integral od g(x) konacan? Koje su to tvrdnje koje su spomenute u rjesenjima?


[Vrh]
Gost






PostPostano: 10:37 sri, 19. 6. 2013    Naslov: Citirajte i odgovorite

[quote="Anonymous"]http://web.math.pmf.unizg.hr/nastava/mii/files/mii_2013_kol2_rj.pdf

Nas kolokvij, ZAD 4, (b)

Moze li objasnjenje cijelog zadatka, tj. dio gdje smo trazili dominantnu fju, zasto se x gledao po intervalima te kako smo zakljucili da je integral od g(x) konacan? Koje su to tvrdnje koje su spomenute u rjesenjima?[/quote]

Zanmarite drugi dio pitanja za konacnost integrala.
Anonymous (napisa):
http://web.math.pmf.unizg.hr/nastava/mii/files/mii_2013_kol2_rj.pdf

Nas kolokvij, ZAD 4, (b)

Moze li objasnjenje cijelog zadatka, tj. dio gdje smo trazili dominantnu fju, zasto se x gledao po intervalima te kako smo zakljucili da je integral od g(x) konacan? Koje su to tvrdnje koje su spomenute u rjesenjima?


Zanmarite drugi dio pitanja za konacnost integrala.


[Vrh]
Lafiel
Forumaš(ica)
Forumaš(ica)


Pridružen/a: 26. 09. 2007. (09:56:59)
Postovi: (153)16
Spol: žensko
Sarma = la pohva - posuda
27 = 51 - 24

PostPostano: 19:29 čet, 20. 6. 2013    Naslov: Citirajte i odgovorite

Pitanje vezano uz dokaz [url=http://web.math.pmf.unizg.hr/nastava/mii/files/mii_predavanja.pdf]Lebesgueove kompozicije[/url] (zadnja strana skripte): Kako znamo da je [tex]\nu_s \perp \mu[/tex] ?
Pitanje vezano uz dokaz Lebesgueove kompozicije (zadnja strana skripte): Kako znamo da je [tex]\nu_s \perp \mu[/tex] ?



_________________
Weit von hier fällt Gold von den Sternen
[Vrh]
Korisnički profil Pošaljite privatnu poruku
Gost






PostPostano: 20:32 čet, 20. 6. 2013    Naslov: Citirajte i odgovorite

Pogledaj koliko je mi(N) i ni_s(N komplement). Da je prvi nula upravo pokazano, a drugi je 0 jer je tak bas definiran ni_s. Dakle po definiciji su medjusobno singularne.
Pogledaj koliko je mi(N) i ni_s(N komplement). Da je prvi nula upravo pokazano, a drugi je 0 jer je tak bas definiran ni_s. Dakle po definiciji su medjusobno singularne.


[Vrh]
Lafiel
Forumaš(ica)
Forumaš(ica)


Pridružen/a: 26. 09. 2007. (09:56:59)
Postovi: (153)16
Spol: žensko
Sarma = la pohva - posuda
27 = 51 - 24

PostPostano: 20:57 čet, 20. 6. 2013    Naslov: Citirajte i odgovorite

:idea2: Hvala!
Imam ideju! Hvala!



_________________
Weit von hier fällt Gold von den Sternen
[Vrh]
Korisnički profil Pošaljite privatnu poruku
Prethodni postovi:   
Započnite novu temu   Odgovorite na temu   printer-friendly view    Forum@DeGiorgi: Početna -> Kolegiji 3. godine -> Mjera i integral Vremenska zona: GMT + 01:00.
Idite na Prethodno  1, 2, 3, 4, 5  Sljedeće
Stranica 4 / 5.

 
Forum(o)Bir:  
Možete otvarati nove teme.
Možete odgovarati na postove.
Ne možete uređivati Vaše postove.
Ne možete izbrisati Vaše postove.
Ne možete glasovati u anketama.
You cannot attach files in this forum
You can download files in this forum


Powered by phpBB © 2001, 2002 phpBB Group
Theme created by Vjacheslav Trushkin
HR (Cro) by Ančica Sečan